Standard solar theory vs the Birkeland solar model.

Status
Not open for further replies.
M

michaelmozina

Guest
<p><BR/>Replying to:<BR/><DIV CLASS='Discussion_PostQuote'>Neon? You think the Sun's photosphere is made of neon?&nbsp; What gives you that idea, given that nowhere near that amount is found in actual observations?&nbsp; I trust you are not relying on some sort of *model* instead of empirical science.&nbsp; Surely, given your utter disdain for anything other than empirical evidence, you would not reach that conclusion if you didn't have some empirical evidence to back it up.</p><p>I would be very interested to know what that is, and why it contradicts the spectral data so dramatically.</DIV></p><p>Let me begin to answer your questions by talking about spectral data and why the Birkeland model varies from standard theory as it relates to composition.</p><p>A Birkeland solar model is predictated upon the assumption of a separatation of elements.&nbsp; It assumes that any heavy elements would sink, and any lighter elements would rise to the surface/atmosphere.&nbsp; Standard solar theory presumes that all the elements stay "mixed together", and that the surface of the photosphere is composed of a homogenous mixture of these various elements.&nbsp; Note here that planets certainly don't emulate standard theory, but standard theory presumes all the elements stay mixed, iron with hydrogen, nickel with helium, etc. &nbsp;</p><p>This difference in viewpoint has a significant impact on spectral data and how one might "interpret" it.&nbsp; For instance, if our spectral equipment was very limited, but we were able to observe a light on the ocean, we might look at the contents of that light and note the following. There is tunstun in output.&nbsp; There is oxygen in the spectral data, and there is carbon dioxide in the spectrial data and nitrogan as well. &nbsp; What would we learn from that spectral data about the composition of the boat that contains the light we see?&nbsp;&nbsp;</p><p>In a mass separated solar model, hydrogen forms the outer layer because it is the lightest of the elements.&nbsp; It is "hot" compared to the rest of the solar atmosphere because that is where most of the "electrical reconnection"/"particle reconnection" is taking place.&nbsp; Since the hydrogen corona is hottest, it emits the most photons.&nbsp; Since the helium layer (chromosphere) is cooler than the corona, it emits less light, but it emits many more photons that cooler layers that sit under the chromophere. In such a mass separated and heat separated model, we would expect to see an abundance of hydrogen and helium photons.&nbsp; If you simply counted the the number of photons present, and the blocking effect of the upper atmosphere, you might get the mistaken impression that the sun is mostly made of hydrogen and helium, especially if you assume everything is homogenously mixed at the photosphere.</p><p>So why double layers of mass separated plasma?&nbsp; Plasma has very predictable and observed properties in the presense of electrical current. It tends to form "double layers" inside the plasma to isolate some (charged) areas from other areas.&nbsp;&nbsp; Any EU oriented solar theory is likely to assume some separation of elements, and in fact this particular solar model predicts concentric double layers will form and will result in the separation of elements by atomic weight. </p><p>Ok, so why is the photosphere mostly Neon?&nbsp; As current is applied to the solar atmosphere (like any plasma), a separation of the elements takes place and concentric double layers of plasma form around the sun.&nbsp; We already know that there are at least three outer layers of the sun, the photosphere, the chromosphere and the corona.&nbsp;&nbsp; Why are these layers there according to standard solar theory?&nbsp; In EU theory (or at least my brand of EU theory) they exist because each element forms it own current carrying double layer.</p><p>So how did I decide which elements were present?&nbsp; I sat down and looked at the SERTS (spectral) data to find out which elements were present.&nbsp; I noted which ion emissions were present, and I noted which ones "changed" during heavy solar activity.&nbsp; In the SERTS data we find lots of iron ions, lots of nickel ions, lots of calcium, various ions of neon, and silicon, etc.&nbsp; I then looked at the atomic weight of each element present and arranged them into double layers by their atomic weight and thought about the implications of a heat separated/mass separated model and how it "look" from a spectral point of view.&nbsp; The fact that the outer part of the atmosphere is mostly made of hydrogen and helium, and the fact these are the hottest layers of the atmosphere causes these elements to emit and block the most photons.&nbsp; The spectral data will necessarily be skewed in that direction.&nbsp; The fact that these elements emit the most photons and block the most photons does not mean they are the most abundant elements present in a mass separated solar model. It simply means they emit the most photons due to the heat in the upper atmosphere, and they block the most photons from below because they represent the upper atmosphere of the sun.</p><p>I looked at the coronal loops and assumed they contained and emitted most of the iron and nickel ion photons, I assumed the crust contained most of the heavy elements, and I arranged the rest of the SERTS elements by atomic weight.&nbsp; This implies that the outer layer (corona) is mostly hydrogen, the chromosphere is mostly helium, the photosphere is mostly neon, and the layer under the photosphere is mostly silicon.&nbsp; Under that probably sits a layer of calcium as well.</p><p>The key from my perspective came in sunspot activity when I realized that the photosphere was made of a different material than the layer underneath of it.&nbsp; All along the edges of the sunspot I could see that light ended at a specific depth.&nbsp; In other words the photosphere is simply an atmospheric layer, like a cloud cover over a planet.&nbsp; During a hurricane, you can sometimes see under the cloud cover.&nbsp; Likewise during sunspot activity, you can see under the photosphere to another layer below, one that does not emit anywhere near the same amount of visible light, one that begins at a very specific depth under the photosphere.&nbsp;&nbsp; The fact I could not see light from along the edges of the umbra was the key observation. </p><p>Replying to:<BR/><DIV CLASS='Discussion_PostQuote'>I remain baffled that you exclude observational evidence from empirical science.&nbsp; Your responses above did not help me understand why you discount it.&nbsp; I find myself wondering if this strange distinction is being drawn purely because you don't like what observational evidence has found.</DIV></p><p>Quite the contrary.&nbsp; It bothered me emensly at first that the the spectral data contained such an over abundance of hydrogen and helium.&nbsp; I struggled with that issue for some time before fully realizing the implications of what I was observing and the model I was slowly coming to.&nbsp; It was important to me that my model satisfy all the observational evidence, not just the satellite evidence.&nbsp;&nbsp; Another turning point for me was finding Dr. Manuels work.&nbsp; The final straws however were the discovery of a "stratification subsurface" by Dr Alexander Kosovichev, and the discovery of Kristian Birkeland's work with terellas in a vacuum that ultimately convinced me of the validity of this model.&nbsp; The satellite data was not enough.&nbsp; It was a good start IMO, but every solar model should account for every observed bit of data. Birkeland's solar model can account for these observations.&nbsp; I've yet to hear anyone even explain the first satellite image on my website using standard solar theory,let alone the heliosiesmology data, or the coronal loops, or the acceleration of solar wind particles or any of the things that Birkelands solar model explains quite well.&nbsp; From my perspective at least, this is all about emprical science.</p><p>Replying to:<BR/><DIV CLASS='Discussion_PostQuote'>I also find it interesting that after my example about the neutrino experiment, you shifted from "astronomy does no experimentation" to "astronomy does a study particles", and after I suggested looking for experiments involving electricity, you shifted to "96% of astronomy doesn't use experimentation".&nbsp; You're realizing that I called you on your hyperbole, aren't you? </DIV></p><p>Yes, I fully realize this fact.&nbsp; That is why I admitted that I painted the industry with too broad of a brush.&nbsp; I fully admit that what your brother does for a living falls under what I would call "real physics" (because it involves controlled physical experimentation), where the study of "inflation"" is metaphysical in nature because it lacks such experimentation. </p><p>What I was trying to point out however is that the industry is used to mathematically modeling things like "dark energy", and "dark matter", "inflation"" and "magnetic reconnection" without ever demonstrating that any of them exist in nature.&nbsp; The LAMBDA-CDM theory (current popular) presumes that baryonic mass makes up only 4% of nature.&nbsp;&nbsp; It is predictated upon "dark matter", and "dark energy" and metaphysicsal entities that have not been shown to exist in nature.&nbsp; In that sense, 96% of standard theory is metaphysical in nature.&nbsp; Now there may be some astronomers like your bother that study the 4% of the universe they can demonstrate is physical in nature, and therefore my statements may not apply to 4% of the astronomers in the industry.&nbsp; On the other hand I see lots of folks writing papers about "dark energy", "dark matter", "inflation", "magnetic reconnection", etc that have never ever been shown to actually exist in nature.&nbsp; These folks are used to simply stuffing the gaps of their metaphysical entities with some math, some nice computer models.&nbsp; It is common practice to computer model these things even though they have never been shown to actually have any effect on any phyical thing in nature.&nbsp; That's the equivelant of mathematically modeling magic elves from my perspective.&nbsp; The was the point I was trying to make.&nbsp; Yes I know I was unfairly painting your brother and other physical scientists that are also into astronomy&nbsp; with too broad of a brush.&nbsp; For that I appologize, but the point is still valid.&nbsp; Current astronomy theory is based on 96% metaphysics, and 4% real physics.&nbsp; Your brother, and other like him study real physical things in real physical experiments.&nbsp; They don't simply program computers to simulate something they can't reproduce in a lab.&nbsp; </p><p>Replying to:<BR/><DIV CLASS='Discussion_PostQuote'>I have no clue where you get that precise number from, or even how one could arrive at a percentage.</DIV></p><p>It comes right from standard Lambda-CDM theory.</p><p>Replying to:<BR/><DIV CLASS='Discussion_PostQuote'>Either way, it seems somewhat amusing that you attempted to indicate that astronomers don't try to do any experiments, particularly on the subject of magnetic reconnection, by posting a link to an experimental study of magnetic reconnection....</DIV></p><p>I only pointed out that it never defined that actual "physics" they claim is unique to "magnetic reconnection" and the "experiment" uses electrons to generate the powerful magnetic fields they use.&nbsp;&nbsp; The fact it's a physical experiment that "claims" to study "magnetic reconnection" does not mean that they have verified it's existence.&nbsp; If they wrote about their experiences in a lab, I would really have no complaint.&nbsp; it's when they point to the sky and claim "magnetic reconnection did it" that I get testy. &nbsp; Show me exactly (physically)&nbsp; how "magnetic reconnection" can be shown to release energy in a unique way (other than electrical and particle reconnection) and then I'll be happy to let anyone point to the sky and claim magnetic reconnection did it.&nbsp; Until they demonstate it's real, it's premature to blame anything in the sky on "magnetic reconnection". The only writing I should see on this subject should not come from astronomers, but from particle physicists in the lab.&nbsp; That's not how it works. &nbsp; The Hinode group keeps making wild claims about "magnetic reconnection" based on a non standard (non Alfven) brand of MHD theory and a some bizarre computer model that allows magnetic fields to change "lines" when in the lab they always form a full continuum.&nbsp; That's what irks me about magnetic reconnection. It is a branch of science that *must* be emprically demonstrate *before* we go pointing at things in the distance and blaming them on "magnetic reconnection". There is obviously an easier way to explain these things using current flow and PPL's "experiments' use "current flow" to operate.&nbsp; If we shut of the controlled variable of electron flow to create the magnetic fields, the whole experiment is a dud.&nbsp; Lets see them explain the physics behind magnetic reconnection.&nbsp; I've never seen it done and then shown to exist in a controlled experiment. I've seen electricity heat up plasma with my own eyes.&nbsp; I know that electrical current can create million degree coronal loops.&nbsp; I have no fatih that magnetic reconnection is anything other than a poorly formed theory that is based on a misconception about electrical and MHD theory.</p><p>Replying to:<BR/><DIV CLASS='Discussion_PostQuote'>Talking of the magnetic reconnection experiment that you linked to (pretty much nuking your argument that those closed-minded astronomers never do any experiements), the webpage says this: "The charged particles which create the aurora are thought to be accelerated through magnetic reconnection."&nbsp; It doesn't say magnetic reconnection is the aurora, as you seemed to be implying.</DIV></p><p>It is suggesting that the acceleration of the solar wind is caused by "magnetic reconnection".&nbsp; Nothing like that is occuring.&nbsp; Birkeland created "aurora" (and coronal loops) with "electron flow" in a real life science experiment. When did these guys ever create an aurora using "magnetic reconnection" in a lab before claiming that aurora were linked to "magnetic reconnection"?&nbsp; I resent those kinds of comments unless they are due to the results of real scientific experiments.&nbsp; Those comments were not the results of such experiments.&nbsp; </p><p>Replying to:<BR/><DIV CLASS='Discussion_PostQuote'>It says that magnetic reconnection is involved -- which was exactly what I was getting at!&nbsp; The aurora is caused by charged particles hitting the upper atmosphere, same as what Birkeland demonstrated years ago.</DIV></p><p>His experiments used *external* electrical current to create these fields, namely in the form of cathode rays. A relatively negatively charged heliopshere would explain why we see solar wind acceleration from all around the sun.&nbsp; A couple of odd ball "magnetic reconnection" events doesn't explain that behavior. Why then would I believe that magnetic reconnection has anything to do with acceleration solar wind particles or aurora?</p><p>Replying to:<BR/><DIV CLASS='Discussion_PostQuote'>Magnetic reconnection is involved in getting them to the right spot and in the right way to make the upper atmosphere glow.&nbsp; Honestly, if you don't even understand what they're saying about magnetic reconnection, it's no wonder you think it's all just BS.</DIV></p><p>I'm not alone, and it's not from a lack of education on the subejct, quite the contrary.&nbsp; Alfven thought it was BS too. I'm afraid I'm a bit "too" educated on MHD theory and electrical theory to buy that song and dance routine.&nbsp;</p><p>According to the theory, a few lines of magnetism might cross and toss out some particles.&nbsp; So what?&nbsp; It would have to occur on an ongoing basis from everwhere around the sun, all the time for such a thing to have any significant effect on solar wind.&nbsp; Such a process should be extremely common and easy to simulate in a lab.&nbsp; I know for a fact that Birkeland was able to do this with electricity.&nbsp; I've know for fact they can't do this in a lab with "magnetic reconnection" since the whole process is based on sub-second bursts of electron flow. &nbsp; Even their experiment is dead in the water without electron flow.&nbsp; I see nothing to suggest that charged particle acceleration and solar wind acceleration in particular has anything at all to do with "magnetic reconnection". This is pure hype and sales pitch material as best as I can tell.</p><p>Replying to:<BR/><DIV CLASS='Discussion_PostQuote'>But I have to wonder how you could be demonstrating such intelligence and such erudition without understanding what it is these people are actually saying -- unless you are allowing your bias to seriously influence you, which is pretty rich considering that this whole thread is about your believe that there is a conspiracy (or at least a serious bias) to suppress the electric universe theory.</DIV></p><p>From my vantage point you have this backwards.&nbsp; It is because I know a bit too much about MHD theory, Alfven's views on "magnetic reconnection" theory, I know too much about electrical theory and magnetic fields to buy this concept without emprical support.&nbsp; If I was blissfully ignorant to these issues, I might easily buy what they claim without so much as a second thought.&nbsp; It is only because I've actually read Alfven's book "Cosmic Plasma", which most astronomers have never read, that I find these ideas to be questionable.&nbsp; I'm not asking for the moon.&nbsp; I'm simply asking these folks to emprically demonstrate thair beliefs in a lab.&nbsp; How hard can it be if "magnetic reconnection" is so common in nature, and it accelerates solar wind constantly, and regularly from all around the sun?&nbsp; Why is the onus of responsibility on me to believe them rather than the onus of responsibility on them to demonstrate their point physicallly, and describe the phyiscs involved in the energy release process?&nbsp; Birkeland did not have any trouble showing his work in an emprical experiment. Why aren' they required to reproduce aurora using magnetic reconnection power before claiming that magnetic reconnnection is in any way asociated with solar wind or aurora activity?&nbsp;</p> <div class="Discussion_UserSignature"> It seems to be a natural consequence of our points of view to assume that the whole of space is filled with electrons and flying electric ions of all kinds. - Kristian Birkeland </div>
 
O

origin

Guest
<p><BR/>Replying to:<BR/><DIV CLASS='Discussion_PostQuote'>&nbsp; Alfven thought it was BS too.</DIV></p><p>Alfven is dead move on..&nbsp;</p><p>Replying to:<BR/><DIV CLASS='Discussion_PostQuote'>I'm afraid I'm a bit "too" educated on MHD theory </DIV></p><p><img src="http://sitelife.space.com/ver1.0/content/scripts/tinymce/plugins/emotions/images/smiley-laughing.gif" border="0" alt="Laughing" title="Laughing" />&nbsp;</p><p>Replying to:<BR/><DIV CLASS='Discussion_PostQuote'>I'm not asking for the moon.&nbsp; I'm simply asking these folks to emprically demonstrate thair beliefs in a lab.&nbsp; How hard can it be if "magnetic reconnection" is so common in nature, and it accelerates solar wind constantly, and regularly from all around the sun?&nbsp; Why is the onus of responsibility on me to believe them rather than the onus of responsibility on them to demonstrate their point physicallly, and describe the phyiscs involved in the energy release process?&nbsp;&nbsp; <br />Posted by michaelmozina</DIV></p><p>Hmm, an experiment on Magnetic Reconnection.&nbsp; Would this do? </p><p>http://mrx.pppl.gov/&nbsp; This site is called the Magnetic Reconnection Experiment.</p><p>They have results and everything.&nbsp; These guys even used math like integration and stuff.&nbsp; No metaphysics here, just good old fashion roll up your sleeves empirical data.</p><p>Does this qualify as good data for you.&nbsp; I am guessing this does not qualify as good data - certainly not to the level of Birkelands metal ball experiments.</p><p>How do I know this does not qualify as acceptable?&nbsp; Because it does not fit with what you want to be true.</p><p>Luckily,&nbsp;you&nbsp;will not let facts and data confuse the issue,&nbsp;you will go on blithely believing your fantasies which is just fine, but I worry that your BS may confuse a young impressionable person and have them waste their time on this silly,&nbsp;fruitless conjecture.</p><p><br /><em>edited to fix formating</em><br />&nbsp;</p> <div class="Discussion_UserSignature"> </div>
 
D

DrRocket

Guest
<p><BR/>Replying to:<BR/><DIV CLASS='Discussion_PostQuote'>&nbsp;... The Hinode group keeps making wild claims about "magnetic reconnection" based on a non standard (non Alfven) brand of MHD theory and a some bizarre computer model that allows magnetic fields to change "lines" when in the lab they always form a full continuum.&nbsp;...&nbsp; <br />Posted by michaelmozina</DIV></p><p>You have brought up tis pont repeatedly, but I think I am only now beginning to understand what you mean.&nbsp; And it is based on misconceptions of both electrodynamics and what physicists and engineers really think.</p><p>&nbsp;I think you are under the impression that when physicista&nbsp;and electrical engineers&nbsp;talk about magnetic field line, that they are saying that the magnetic force is felt only along those lines and that no force is felt between the lines.&nbsp; Such is not the case.&nbsp; A magnet field is a vector field, to each point in space is assigned a vector that represents the magnitude and direction of a quantity that describes the magnetic force on a moving charge.&nbsp; That field is usually continuous in both magnitude and direction, and in that sense forms a coninuum.&nbsp; The magneteic field lines are merely lines to which the field vectors are tangent and are drawn so that the density, or spacing, increases to indicate increasing magnitude of the vectors.&nbsp; The lines are a visualization tool, useful to both physicists and engineers.&nbsp; They allow one with some knowledge of electromagnetism to draw and read a figure describing the field.&nbsp; Since in reality the field exists in 3-space and the magnetic field also consists of 3-vectors, to truly produce a graph of the magnetic field you would need a 6-dimensional piece of paper.&nbsp; Looking at a plane slice and even assuming that the magnetic field is confined to that plane, a true graph of the field would require 4 dimensions.&nbsp; Lacking the necessary multi-dimensional&nbsp;paper, physicists and engineers draw field lines.&nbsp;</p><p>&nbsp;</p> <div class="Discussion_UserSignature"> </div>
 
M

michaelmozina

Guest
<p><BR/>Replying to:<BR/><DIV CLASS='Discussion_PostQuote'>Alfven is dead move on..</DIV></p><p>What does that have to do with anything?&nbsp; He still wrote MHD theory.&nbsp; Do you have a Nobel prize yet by the way?&nbsp; Should I ignore that too?&nbsp; Didn't he say "magnetic reconnection" was psuedoscience?</p><p>Replying to:<BR/><DIV CLASS='Discussion_PostQuote'> Hmm, an experiment on Magnetic Reconnection.&nbsp; Would this do? http://mrx.pppl.gov/ This site is called the Magnetic Reconnection Experiment.They have results and everything. </DIV></p><p>Then surely you can point me to the paper that describes the physical (not metaphysical) energy release process that is *unique* to "magnetic reconnection" and shows how that isolated that signature from other particle and electrical 'reconnections" in plasma?</p><p>Replying to:<BR/><DIV CLASS='Discussion_PostQuote'>These guys even used math like integration and stuff.&nbsp; </DIV></p><p>You can apply all the integration to elves you like, but it won't make them "real".&nbsp; Only a controlled scientific experiment can demonstrate something exists in reality.&nbsp; Math by itself won't cut it.</p><p>Replying to:<BR/><DIV CLASS='Discussion_PostQuote'>No metaphysics here, just good old fashion roll up your sleeves empirical data.</DIV></p><p>What empirical data?&nbsp; What is the unique physical energy release signature of "magnetic reconnection"?&nbsp; How is it different from ordinary particle and electrical interactions in plasma? </p><p>Replying to:<BR/><DIV CLASS='Discussion_PostQuote'>Does this qualify as good data for you.&nbsp; I am guessing this does not qualify as good data - certainly not to the level of Birkelands metal ball experiments.</DIV></p><p>Did they create a continious aurora around a sphere in a vacuum using "magnetic reconnection" as an energy source?&nbsp;&nbsp;</p><p>Replying to:<BR/><DIV CLASS='Discussion_PostQuote'>How do I know this does not qualify as acceptable? </DIV></p><p>Because they didn't isolate the unique physics behind "magnetic reconnection" perhaps?&nbsp; Maybe because they never created stable aurora with "magnetic reconnection" as a power source of energy perhaps? &nbsp; </p><p>Replying to:<BR/><DIV CLASS='Discussion_PostQuote'>Because it does not fit with what you want to be true.</DIV></p><p>It doesn't fit with what Alfven believed to be true either.&nbsp; What do I personally have to do with emprical science.&nbsp; Emprical science is pretty cut and dry.&nbsp; Birkeland had no trouble creating aurora around spheres in a lab using external electricity as the power source.&nbsp; When was that done with "magnetic reconnection"?</p><p>Replying to:<BR/><DIV CLASS='Discussion_PostQuote'>Luckily,&nbsp;you&nbsp;will not let facts and data confuse the issue,&nbsp;you will go on blithely believing your fantasies which is just fine, but I worry that your BS may confuse a young impressionable person and have them waste their time on this silly,&nbsp;fruitless conjecture.edited to fix formating&nbsp; <br /> Posted by origin</DIV></p><p>I'm hoping that some impressionable young men and women watch people like you lash out at real physicsl like MHD theory in irrational ways, and they'll see the irrational predjudice for what it is.&nbsp; I hope they also learn a little something about EU theory along the way, and they keep an open mind.&nbsp; That's what I'm shooting for.&nbsp; I could care less what you think of EU theory or Birkeland's solar model because you aren't interested in the truth.&nbsp; If you were, you'd sit down and carefully explain the physics behind "magnetic reconnection' and what makes it a unique energy release mechanism.&nbsp; You'd explain that first image on my website in careful detail.&nbsp; You won't do these things becaue they take time and effort and attention to scientific detail.&nbsp; It's easy for folks without competence in a scienfific field to attack an individual.&nbsp; It's much harder to address the scientific issues head on.&nbsp; If you believe Alfven (a guy with a Nobel prize) was wrong about "magnetic reconnection", then carefully explain to us what the unique form of energy release behind "magnetic reconnection" is all about.&nbsp; Tell us how it is unique and different from particle reconnection and electrical reconnection, and how this difference was tested for in controlled conditions.&nbsp; Leave nothing to chance here.&nbsp; Lay it out for us since you're so smart, and I'm so dumb.</p><p>&nbsp;</p> <div class="Discussion_UserSignature"> It seems to be a natural consequence of our points of view to assume that the whole of space is filled with electrons and flying electric ions of all kinds. - Kristian Birkeland </div>
 
M

michaelmozina

Guest
<p><BR/>Replying to:<BR/><DIV CLASS='Discussion_PostQuote'>You have brought up tis pont repeatedly, but I think I am only now beginning to understand what you mean. </DIV></p><p>I'm not sure you quite understand my concern, though I appreciate the very detailed (and very clear explanation) behind a full dimensional magnetic field continuum.&nbsp;&nbsp; Here is my concern.</p>http://en.wikipedia.org/wiki/Magnetic_reconnection<p>Replying to:<BR/><DIV CLASS='Discussion_PostQuote'> <strong>Magnetic reconnection</strong> is the process whereby magnetic field lines from different magnetic domains are spliced to one another, changing their patterns of connectivity with respect to the sources. It is a violation of an approximate conservation law in plasma physics, and can concentrate mechanical or magnetic energy in both space and time.</DIV></p><p>Conservation of energy laws are "laws" for a reason.&nbsp; Magnetic reconnection theory is predictated upon two "continuum" of magnetic fields releasing energy by their magnetic interactions.&nbsp; Particles could interact and release energy, certainly flowing charged particles of current carrying plasma could release energy like that.&nbsp; Magnetic fields alone&nbsp; however cannot ever do that because they form a full continuum. They don't "release energy" in this way. It would in fact be a violation of the conservation of energy laws if they did. &nbsp;</p><p>IMO the whole theory behind "magnetic reconnection" is based on an ignorance of the electrical nature of plasma, the kinetic energy found in the charged particle movement, and a basic ignorance of the all pervasive field that sustains them.&nbsp; There is no violation of energy laws going on in the upper atmosphere of the sun. There are simply very powerful electrical discharges traversing the solar atmosphere.&nbsp; If one is ignorant of the current flow that sustains this process, and percieves it to be a completely internal process, then it may "seem" like this is a magneticly driven event, but it's an electrically driven event as Birkeland demonstrated over 100 years ago.&nbsp; There is not such thing as "magnetic reconnection".&nbsp; Continuums don't just swap magnetic lines like that and release energy in the process.&nbsp; Only current flow and charged particles will release energy, and there is no concervation law being violated in that process. </p> <div class="Discussion_UserSignature"> It seems to be a natural consequence of our points of view to assume that the whole of space is filled with electrons and flying electric ions of all kinds. - Kristian Birkeland </div>
 
O

origin

Guest
<p><BR/>Replying to:<BR/><DIV CLASS='Discussion_PostQuote'>What empirical data?&nbsp; What is the unique physical energy release signature of "magnetic reconnection"?&nbsp; How is it different from ordinary particle and electrical interactions in plasma? Did they create a continious aurora around a sphere in a vacuum using "magnetic reconnection" as an energy source?&nbsp;&nbsp;Because they didn't isolate the unique physics behind "magnetic reconnection" perhaps?</DIV></p><p>Well I'll be the experimental&nbsp;data is no good!&nbsp; Gee&nbsp;whoda thought.&nbsp; Lets see the why is the experiment no good:</p><p>1.&nbsp; <font color="#0000ff">What is the unique physical energy release signature of the 'Magnetic Reconnection'?</font>&nbsp; That does not have anything to do with whether magnetic reconnection is pseudo-science or not.&nbsp;&nbsp;Guess it was time to move&nbsp;the goal posts. </p><p>2.&nbsp;&nbsp;<font color="#0000ff">How is it different than ordinary interactions?</font>&nbsp; You didn't look at the results did you.&nbsp; Didn't want that nasty data confusing the issue, huh?</p><p>3.&nbsp; <font color="#0000ff">Didn't make an aurora around a sphere.</font>&nbsp; True,&nbsp;I didn't know that was a requirement to show that magnetic reconnection wasn't pseudo-science.&nbsp; Move the goal posts much?&nbsp;</p><p>4.&nbsp; <font color="#0000ff">Because they didn't isolate the physics behind "magnetic reconnection".</font>&nbsp; I don't even know what the heck this is suppose to mean.&nbsp; I suppose you mean using mathematics to show exactly what is physically going on - in which case you would say it is mathematical mumbo-jumbo.</p><p>You are so transparent, Michael....</p><p>Replying to:<BR/><DIV CLASS='Discussion_PostQuote'>Lay it out for us since you're so smart, and I'm so dumb.</DIV>&nbsp; </p><p>Actually this is the saddest part - I don't think you are dumb, I think you are obsessed with this stupid conjecture and you are using all of your intellect to support an unsupportable position.&nbsp; What a freaking waste!<img src="http://sitelife.space.com/ver1.0/content/scripts/tinymce/plugins/emotions/images/smiley-frown.gif" border="0" alt="Frown" title="Frown" /></p><p><br />Replying to:<BR/><DIV CLASS='Discussion_PostQuote'>Posted by michaelmozina</DIV><br /></p> <div class="Discussion_UserSignature"> </div>
 
D

DrRocket

Guest
<p><BR/>Replying to:<BR/><DIV CLASS='Discussion_PostQuote'>I'm not sure you quite understand my concern, though I appreciate the very detailed (and very clear explanation) behind a full dimensional magnetic field continuum.&nbsp;&nbsp; Here is my concern.http://en.wikipedia.org/wiki/Magnetic_reconnectionConservation of energy laws are "laws" for a reason.</p><p><font color="#0000ff">Yes.&nbsp; But&nbsp;I think the "approximate conservation law&nbsp;in plasma physics" that is broken is magnetic topology conservation, rather that conservation of energy per se. http://www.astrophys-space-sci-trans.net/2/63/2006/astra-2-63-2006.pdf</font></p><p>&nbsp; Magnetic reconnection theory is predictated upon two "continuum" of magnetic fields releasing energy by their magnetic interactions.&nbsp; Particles could interact and release energy, certainly flowing charged particles of current carrying plasma could release energy like that.&nbsp; Magnetic fields alone&nbsp; however cannot ever do that because they form a full continuum. They don't "release energy" in this way. It would in fact be a violation of the conservation of energy laws if they did. </p><p><font color="#0000ff">Electromagnetic fields are at a fundamental level just a bunch of photons,&nbsp;pure energy.&nbsp; They can and do release or impart energy.&nbsp; That is exactly what happens in a common generator -- magnetic fields cause electrons, massive particles, to move and thereby have kinetic energy.&nbsp; A changing magnetic flux induces a current by pushing on charged particles.&nbsp; Magnetic reconnection creates a very rapidly changing magnetic field, and a&nbsp; plasma is just a bunch of charged particles. </font></p><p>&nbsp;</p><p>&nbsp;IMO the whole theory behind "magnetic reconnection" is based on an ignorance of the electrical nature of plasma, the kinetic energy found in the charged particle movement, and a basic ignorance of the all pervasive field that sustains them.&nbsp;</p><p><font color="#0000ff">You could not possibly be more wrong.&nbsp; Itis based on a very profound knowledge of the electrical (ionized) nature of the constituents of a plasma which feel the electromagnetic force and participate in it, in part by gaining or losing kinetic energy in response to an interaction with the pervasive electromagnetic field that.&nbsp; The particles not only are affected by the field, they create it through their motion.</font></p><p>There is no violation of energy laws going on in the upper atmosphere of the sun.</p><p><font color="#0000ff">On a macro level this is correct.&nbsp; However, quantum mechanics becaue of&nbsp;the uncertainty principle does permit momentary (very short) processes that do violate conservation of energy.&nbsp; </font>&nbsp;</p><p>&nbsp;</p><p>There are simply very powerful electrical discharges traversing the solar atmosphere.</p><p><font color="#0000ff">I'm not quite sure what you mean here.&nbsp; There are a lot of things&nbsp;going on that involve the movement of charged particles, so I agree in that sense.&nbsp; But if you are referring to your notion of&nbsp;some&nbsp;large current flow through the cosmos&nbsp;and providing the primary energy source for the sun, as you have stated elsewhere, then you are all wet.</font></p><p>&nbsp;If one is ignorant of the current flow that sustains this process, and percieves it to be a completely internal process, then it may "seem" like this is a magneticly driven event, but it's an electrically driven event as Birkeland demonstrated over 100 years ago.&nbsp;</p><p><font color="#0000ff">Yep.&nbsp; Soaked to the skin, drowning and don't even know it.&nbsp;As I pointed out elsewhere if there were such a current powering&nbsp;the Sun, a&nbsp;very conductive body as we both agree, the current required would be enormous and the resulting magnetic field would swamp the observed magnetic field at the surface of the Earth.</font>&nbsp;</p><p>&nbsp;There is not such thing as "magnetic reconnection".&nbsp; Continuums don't just swap magnetic lines like that and release energy in the process.&nbsp; Only current flow and charged particles will release energy...</p><p>&nbsp;<font color="#0000ff">This is an unsubstantiated assertion with no basis in either physics or mathematics.&nbsp; And don't try to wriggle out of this with another "Alfven said".&nbsp; A lot has been learned since he did his work.</font></p><p>, and there is no concervation law being violated in that process. </p><p><font color="#0000ff">A&nbsp;glimmer of truth.&nbsp; </font><br />Posted by michaelmozina</DIV><br /></p> <div class="Discussion_UserSignature"> </div>
 
M

michaelmozina

Guest
<p><BR/>Replying to:<BR/><DIV CLASS='Discussion_PostQuote'>Well I'll be the experimental&nbsp;data is no good!&nbsp; Gee&nbsp;whoda thought.&nbsp; Lets see the why is the experiment no good:1.&nbsp; What is the unique physical energy release signature of the 'Magnetic Reconnection'?&nbsp; That does not have anything to do with whether magnetic reconnection is pseudo-science or not.&nbsp;&nbsp;Guess it was time to move&nbsp;the goal posts.</DIV></p><p>Boloney.&nbsp; It's the key issue, and the 64K question which is why you are running from it.</p><p>From Wiki&nbsp;</p><p>Replying to:<BR/><DIV CLASS='Discussion_PostQuote'>Magnetic Reconnection: This view is a cross-section through four magnetic domains undergoing separator reconnection. Two separatrices divide space into four magnetic domains with a separator at the center of the figure. Field lines (and associated plasma) flow inward from above and below the separator, reconnect, and spring outward horizontally. A current sheet (as shown) may be present but is not required for reconnection to occur. This process is not well understood: once started, it proceeds many orders of magnitude faster than predicted by calculation from first principles.</DIV></p><p>Show me where energy release process called "magnetic reconnection"" was ever performed in a lab *without* a current sheet.&nbsp; Don't tell me PPL did it, because their whole experiment is designed to create them.</p><p>Replying to:<BR/><DIV CLASS='Discussion_PostQuote'>2.&nbsp;&nbsp;How is it different than ordinary interactions?&nbsp; You didn't look at the results did you.&nbsp; Didn't want that nasty data confusing the issue, huh?</DIV></p><p>Yes, I did see their results, and I know that they never did isolate the actual "physics" (not metaphysics) behind this conservation of energy defying concept.&nbsp; They certainly never defied any conservation of energy laws since they use electricty to power the entire experiment.</p><p>Replying to:<BR/><DIV CLASS='Discussion_PostQuote'>3.&nbsp; Didn't make an aurora around a sphere.&nbsp; True,&nbsp;I didn't know that was a requirement to show that magnetic reconnection wasn't pseudo-science.</DIV></p><p>If they are going to claim that aurora are in any way linked to "magnetic reconnection", I want to see them produce them with "magnetic reconnection" like Birkeland did with "electrictiy".&nbsp; Otherwise they are simply misleading those impressionable young minds into believing things that are simply untrue.</p><p>Replying to:<BR/><DIV CLASS='Discussion_PostQuote'>Move the goal posts much?&nbsp;4.&nbsp; Because they didn't isolate the physics behind "magnetic reconnection".&nbsp; I don't even know what the heck this is suppose to mean.</DIV></p><p>It's almost sad that you don't know what that means and why I ask it.&nbsp; If there is such a thing as "magnetic reconnection", lets see them explain the physics behind it.&nbsp; I know that charge particles inside a current stream slamming into one another will release all sorts of kinetic and EM energy.&nbsp; I've never seen that done with "magnetic reconnection", and I have no idea what they think is releaseing energy exactly.&nbsp; For all I know they simply don't comprehend the fact that particle reconnection and electrical reconnection is occuring inside a current sheet.&nbsp; I have no idea why they believe in a theory that violates conservation of enegy laws, and so I have nothing "physical" to work with.&nbsp; It's not physics, till you or they can explain what is specifically unique about "magnetic reconnection" as an energy source.&nbsp; As far as I know the whole theory is psuedoscience, just as Alfven suggested. and just as the *laws* of physics would insist.</p><p>Replying to:<BR/><DIV CLASS='Discussion_PostQuote'>I suppose you mean using mathematics to show exactly what is physically going on - in which case you would say it is mathematical mumbo-jumbo.You are so transparent, Michael....&nbsp; Actually this is the saddest part - I don't think you are dumb, I think you are obsessed with this stupid conjecture and you are using all of your intellect to support an unsupportable position.&nbsp; What a freaking waste! <br /> Posted by origin</DIV></p><p>Likewise I find most of my conversations with you personally to be a waste of time.&nbsp; You refuse to even consider what I'm saying because your mind it closed.&nbsp; You can't really handle the scientific observations and scientific data, so you spend an inordinate amount of time attacking the individual.&nbsp; That's the part I find the most sad.</p><p>Are you ever going to deal with the physics behind "magnetic reconnection"?&nbsp; How about that first solar satellite image on my website, or maybe you can explain why the solar wind is in a constant state of acceleration?&nbsp; Coronal loops? How do they reach 10's of milions of degree in temperature sitting above a 6K photosphere? </p><p>If one never mentally bothers to address these issues head on in an intellectually honest manner, then it may be possible to remain in a constant state of denial for quite some time.&nbsp; On the other hand, if one puts their nose the grindstone, and really sits down to explain these phenomenon based on the known forces of nature and the *laws of physics*, EU theory looks more promising by the day.&nbsp; </p> <div class="Discussion_UserSignature"> It seems to be a natural consequence of our points of view to assume that the whole of space is filled with electrons and flying electric ions of all kinds. - Kristian Birkeland </div>
 
D

derekmcd

Guest
I don't feel like digging up the quote, but I believe you have previously mentioned something to the effect that the sun's energy output isn't entirely dependent on internal sources.&nbsp; What is the external source of energy? <div class="Discussion_UserSignature"> <div> </div><br /><div><span style="color:#0000ff" class="Apple-style-span">"If something's hard to do, then it's not worth doing." - Homer Simpson</span></div> </div>
 
D

DrRocket

Guest
<p><br />What I have gotten out of your posts on this thread and the previous one is:</p><ul><li>The Sun like the other stars in the universe is primarily powered, not by fusion, but by a current that flows through the cosmos, motive source unknown,&nbsp; Since we do not see the enormous magnetic field at the surface of the Earth that would result fron any such current, one must conclude that Maxwell's equations are very much in error.</li><li>It is electrical effects, rather than gravitational effects, that govern the behavior of the larges masses in the universe at the largest distance scales.&nbsp; Mainstream physicists are just to bliind to see this.</li><li>The photosphere of the Sun is composed&nbsp;primarily of neon, not hydrogen as is believed by the mainstream.&nbsp;</li><li>The Sun is basically a giant neon light bulb.</li><li>The core of the Sun is not a hot gaseous plasma, but rather is solid, primarily composed of nickel and iron or at least a nickel iron shell over a neutron core.&nbsp; It remains solid despite being in the interior of an extremely hot radiating sphere, which would imply an enormous radiant heat flux.</li><li>Magnetic fields are not particularly important to the behavior of plasma at and near the surface of the Sun, but rather it is electrical currents and lightning-like discharges that govern.</li><li>Topological changes in the magnetic field cannot and do not exist, and modern physicists studying the topic are deluded.</li><li>Because physicists and engineers use diagrams with field lines to describe fields, not having at their disposal the necessary 4-dimensional paper to depict them accurately, they don't understand the continuous nature of vector fields.&nbsp; One can only conclude that they have no grasp of the behavior of either magnetic fields or of plasmas.</li><li>Modern physicists, the ongoing experiments at the Princeton Plasma Physics lab notwithstanding, do not have any experimental evidence for their hypotheses regarding magnetic reconnection.&nbsp; The 100 year old work of Birkeland is better and more accurate than what is being done today with far more sophisticated equipment and substantial funding.</li><li>Physicists and astronomers do not understand "electrical theory" as only an electrical engineer can, despite the fact that physics and engineering texts on electromagnetism say the same things in pretty much the same way.</li><li>Mainstream scientists do not give adequate credance to EU theory or solar theory and conspire to prevent its publication, despite the obvious merits.&nbsp; The several papers that have evaluated the premises of EU theory and have found them wanting on scientific grounds should be ignored, as the physicists just don't understand.</li><li>Work such as yours in EU theory and solar physics ought not be ignored.&nbsp; You have the answers to the riddles of the universe.</li></ul><p>All I can say is that I do believe that this sort of thing has received a good deal of consideration in the mainstream of academia.&nbsp; Look up Abnormal Psychology.</p><p>&nbsp;</p> <div class="Discussion_UserSignature"> </div>
 
O

origin

Guest
<p><BR/>Replying to:<BR/><DIV CLASS='Discussion_PostQuote'> If one never mentally bothers to address these issues head on in an intellectually honest manner, then it may be possible to remain in a constant state of denial for quite some time. <br />Posted by michaelmozina</DIV><br /><br />Gee, this is so unbelievably ironic I would think your were making a joke if I didn't know better.</p><p>&nbsp;</p> <div class="Discussion_UserSignature"> </div>
 
S

Saiph

Guest
Boy, you just don't believe in half measures do you Micheal?<BR/><BR/>Okay, I'm going to try to concisely address your posts, and ask you a few key questions.<BR/><BR/>One of my first questions is: Considering just the photospheric layer, and it's spectral emissions, how do you go about calculating the various elemental populations? I'm asking for a brief run down here, such as which equation do you use, what are some things you have to keep in mind.<BR/><BR/>Depending on that answer, I'll consider any compositions you derived from spectra. Otherwise I'll tell you where you went wrong. This approach, btw, is backed by empirical data in our own labs.<BR/><BR/><BR/>Now, as for the seperation of atoms in the sun. Okay, I'll buy that. Heck, I'll give it to you! The standard model already calls for it! Only the standard model doesn't have seperation throughout the entire sun. The core can be heavily seperated. However due to the temperature gradients in the sun, dictated by the fact that it's in hydrostatic equilibrium, and the required energy transportation through the sun to maintain that state...we have a region of the sun with very strong convective currents. This is using the same appraoch to deterimine the flow of boiling water btw. In the presence of such convective currents how do you prevent mixing?<BR/><BR/>-------------------<BR/>The fact that the outer part of the atmosphere is mostly made of hydrogen and helium, and the fact these are the hottest layers of the atmosphere causes these elements to emit and block the most photons. The spectral data will necessarily be skewed in that direction.<BR/>-------------------<BR/><BR/>Here you demonstrate that you have a bit of a clue as to some of the problems in spectroscopy. You correctly indicate that the temperature of the atoms can change how much light they absorb. But you've got something wrong here. You can be to hot, where vast numbers of the atoms are actually stripped of their electrons...and can no longer intercept as much (or in some cases ANY light). here's a small secrete: The suns spectra is heavily dominated by numerous, deep adn dark IRON lines (and other "metals" like calcium), and relatively few helium and hydrogen lines.<BR/><BR/><BR/>-----------------------<BR/>I looked at the coronal loops and assumed they contained and emitted most of the iron and nickel ion photons, I assumed the crust contained most of the heavy elements, and I arranged the rest of the SERTS elements by atomic weight. This implies that the outer layer (corona) is mostly hydrogen, the chromosphere is mostly helium, the photosphere is mostly neon, and the layer under the photosphere is mostly silicon. Under that probably sits a layer of calcium as well.<BR/>------------------------<BR/>First, why are you assuming all this? Look at the spectra and find out. Also, you say that the coronal loops emit mostly iron and nickel spectra, but is mostly hydrogen...<BR/><BR/>????????<BR/>At what point does the sun stop being predominately hydrogen and helium? I'm looking for a "depth" figure here.<BR/><BR/>????????<BR/>You refer to a satelite image nobody's been able to explain...what is the image, I'd love to take a look at it.<BR/><BR/>I see even more references to "overabundance of hydrogen and helium" when that isn't the case. What spectra are you looking at? <div class="Discussion_UserSignature"> <p align="center"><font color="#c0c0c0"><br /></font></p><p align="center"><font color="#999999"><em><font size="1">--------</font></em></font><font color="#999999"><em><font size="1">--------</font></em></font><font color="#999999"><em><font size="1">----</font></em></font><font color="#666699">SaiphMOD@gmail.com </font><font color="#999999"><em><font size="1">-------------------</font></em></font></p><p><font color="#999999"><em><font size="1">"This is my Timey Wimey Detector.  Goes "bing" when there's stuff.  It also fries eggs at 30 paces, wether you want it to or not actually.  I've learned to stay away from hens: It's not pretty when they blow" -- </font></em></font><font size="1" color="#999999">The Tenth Doctor, "Blink"</font></p> </div>
 
C

CalliArcale

Guest
<p>*mod hat on*&nbsp;</p><p>All right, DrRocket, time to scale it back a notch.&nbsp; You're new here, so you don't know that we have a rule against suggesting that one's opponent in a debate has a mental disorder.&nbsp; Besides, in his time here, michaelmozina has demonstrated himself to be a very polite, reasonable individual.&nbsp; indeed, I have seen no one give a better defense of the electric universe theory, and he has generally comported himself in a dignified and respectful manner.&nbsp; We've had to ban a number of EU proponents for their inability to express themselves without insult; he is doing an admirable job of demonstrating that even minority views can be expressed in a reasonable manner.</p><p>Now, I think it behooves those of us representing the majority to demonstrate that we are capable of the same thing.&nbsp; A lot of us are getting our danders up, and that doesn't do any good.&nbsp; Let's all resist the temptation to stoop to cheap shots, okay?</p><p>EDIT:&nbsp; I now notice that michaelmozina did something similar, albeit less obvious, a few posts earlier.&nbsp; So it's coming on both sides.&nbsp; Same reminder applies all around, then.&nbsp; Don't insult your opponents; it's really not neccessary. </p><p>&nbsp;</p><p>*mod hat off*&nbsp;</p> <div class="Discussion_UserSignature"> <p> </p><p><font color="#666699"><em>"People assume that time is a strict progression of cause to effect, but actually from a non-linear, non-subjective viewpoint it's more like a big ball of wibbly wobbly . . . timey wimey . . . stuff."</em>  -- The Tenth Doctor, "Blink"</font></p> </div>
 
C

CalliArcale

Guest
<em>So how did I decide which elements were present?&nbsp; I sat down and looked at the SERTS (spectral) data to find out which elements were present.&nbsp; I noted which ion emissions were present, and I noted which ones "changed" during heavy solar activity.&nbsp; In the SERTS data we find lots of iron ions, lots of nickel ions, lots of calcium, various ions of neon, and silicon, etc.&nbsp; I then looked at the atomic weight of each element present and arranged them into double layers by their atomic weight and thought about the implications of a heat separated/mass separated model and how it "look" from a spectral point of view.&nbsp; The fact that the outer part of the atmosphere is mostly made of hydrogen and helium, and the fact these are the hottest layers of the atmosphere causes these elements to emit and block the most photons.&nbsp; The spectral data will necessarily be skewed in that direction.&nbsp; The fact that these elements emit the most photons and block the most photons does not mean they are the most abundant elements present in a mass separated solar model. It simply means they emit the most photons due to the heat in the upper atmosphere, and they block the most photons from below because they represent the upper atmosphere of the sun.</em><p><em>I looked at the coronal loops and assumed they contained and emitted most of the iron and nickel ion photons, I assumed the crust contained most of the heavy elements, and I arranged the rest of the SERTS elements by atomic weight.&nbsp; This implies that the outer layer (corona) is mostly hydrogen, the chromosphere is mostly helium, the photosphere is mostly neon, and the layer under the photosphere is mostly silicon.&nbsp; Under that probably sits a layer of calcium as well.</em></p><p><em>The key from my perspective came in sunspot activity when I realized that the photosphere was made of a different material than the layer underneath of it.&nbsp; All along the edges of the sunspot I could see that light ended at a specific depth.&nbsp; In other words the photosphere is simply an atmospheric layer, like a cloud cover over a planet.&nbsp; During a hurricane, you can sometimes see under the cloud cover.&nbsp; Likewise during sunspot activity, you can see under the photosphere to another layer below, one that does not emit anywhere near the same amount of visible light, one that begins at a very specific depth under the photosphere.&nbsp;&nbsp; The fact I could not see light from along the edges of the umbra was the key observation.&nbsp;</em></p><p>in other words, you did precisely what you accuse the mainstream scientists of doing -- you took a few observations and then shuffled things around until it fit both the observations and your preconceived notions.&nbsp; Why don't you realize why this makes your accusations of intellectual dishonesty on the part of mainstream astronomers so grating?&nbsp; You are doing precisely what you accuse them of doing.&nbsp; (Which, incidentally, is *not* what they are actually doing.&nbsp; What they are doing is much more involved.&nbsp; However, now that I know your professional background is similar to mine, perhaps that makes sense.&nbsp; Software engineers don't have a whole lot of perspective into the daily workaday life of an astrophysicist.) </p><p>Regarding sunspots, do you think they are dark?&nbsp; They aren't.&nbsp; Only laypeople think they're dark.&nbsp;&nbsp; They're actually extremely luminous.&nbsp; It's just that the surrounding sun is more luminous, so by comparison, they seem dark.</p><p>Watching sunspots through a telescope is fascinating.&nbsp; There was a really big one a couple of years ago, and it was fascinating watching it.&nbsp; You could see the magnetic field lines, plain as day, in the little fringes around the edges.&nbsp; It was just exactly like metal filings around the poles of a magnet.&nbsp; You can see it in the TRACE imagery too, which has awesomely detailed videos of coronal loops.&nbsp; They look exactly like magnetic field lines.</p><p>I still think you've got a very confused impression of what "magnetic reconnection" means.&nbsp; You seem to be using the term to describe a complete model of the Sun, the solar wind, and the Earth's magnetosphere, but the scientists who study the phenomenon are not using the term that way themselves.&nbsp; They are using it to refer to a specific sort of event, not to the entire system. And that may be where the confusion is arising.&nbsp; Certainly it's making it difficult to understand what exactly your objections are to mainstream theories, since so far you seem to be objecting to things which are not claimed by mainstream theory -- such as that energy is created from nothing during magnetic reconnection, which is not claimed at all.&nbsp; When they say energy is released, that implies that energy was stored in some other form, not that it was created out of nothing.</p><p>BTW, when I asked where you could possibly have gotten your "96% of astronomy is metaphysics" statistic, I kinda suspected you'd answer "oh, that's the percentage of dark matter".&nbsp; But that's silly.&nbsp; You were expressing that statistic as a percentage of astronomical empiricism, not as a percentage of how much matter is presently accounted for.&nbsp; Dark matter is certainly not the only thing astronomers study; far from it.&nbsp; More astronomers study things like periodic comets and asteroids and extrasolar planets and planetary nebulas and things like that, which means that the 96% figure is referring to a supposed deficiency in a minority of the discipline, thus grossly distorting the numbers even if the 96% really did refer to percentage of wooiness rather than percentage of matter which has been detected by means other than its gravitational influence. </p> <div class="Discussion_UserSignature"> <p> </p><p><font color="#666699"><em>"People assume that time is a strict progression of cause to effect, but actually from a non-linear, non-subjective viewpoint it's more like a big ball of wibbly wobbly . . . timey wimey . . . stuff."</em>  -- The Tenth Doctor, "Blink"</font></p> </div>
 
C

CalliArcale

Guest
I'll make a larger post over in Missions & Launches on this, but I thought I'd call this to your attention, Michael.&nbsp; NASA has restarted the Solar Probe concept, assigning the development effort to JHU-APL.&nbsp; Solar Probe, hopefully to launch in 2015, will take seven years to adjust its orbit to where it can start the main phase of its mission, lurking behind a carbon composite sunshade so that it can study the near-Sun environment in a way that has never been possible before.&nbsp; I'm very excited, even though we'll have to wait quite a while to see the fruits of this.&nbsp; Unfortunately, getting to the inner solar system is at least as difficult as getting to the outer solar system, because you have to bleed off so much orbital energy to do it usefully.&nbsp; (Early inner solar system missions, like Mariner 10, took much less time to reach their destinations but then went zooming right on by.&nbsp; Solar Probe is aiming for a much more useful orbit, scientifically speaking, so patience will pay off in the end, just as with MESSENGER, which has a similar problem.) <div class="Discussion_UserSignature"> <p> </p><p><font color="#666699"><em>"People assume that time is a strict progression of cause to effect, but actually from a non-linear, non-subjective viewpoint it's more like a big ball of wibbly wobbly . . . timey wimey . . . stuff."</em>  -- The Tenth Doctor, "Blink"</font></p> </div>
 
M

michaelmozina

Guest
<p><BR/>Replying to:<BR/><DIV CLASS='Discussion_PostQuote'>in other words, you did precisely what you accuse the mainstream scientists of doing -- you took a few observations and then shuffled things around until it fit both the observations and your preconceived notions.</DIV></p><p>In a very real sense, yes, that's exactly what I did.&nbsp; That really isn't my primary "beef" however with the mainstream.&nbsp; I don't mind anyone making things "fit" with their preconcieved ideas, as long as they don't try to stuff in a metaphysical force or a particle that has not been shown to exist in nature.</p><p>For instance, hydrogen fusion *could* in fact power a sun.&nbsp; That is not a "metaphysical" idea in any way.&nbsp; Fusion will occur in the right conditions, and therefore it's a valid scientific idea, even if it turns out to be wrong.&nbsp;&nbsp; There is nothing unscientific or metaphysical about just being wrong about a theory.&nbsp;&nbsp; It's when they start trying to make up new forces of nature to stuff the gaps of their understanding that I get squeemish.&nbsp; Fusion is a scientifically viable way to release energy.&nbsp; "Magnetic reconnection" is not.&nbsp; That is because fusion has been demonstrated in controlled conditions, whereas magnetic reconnection has not.&nbsp; I really do not object to most aspects of current solar theory other than "magnetic reconnection". &nbsp; I may not agree with most aspects of current solar theory, but only the magnetic reconnection concept is based on metaphysical claims.&nbsp; I'm not irked by most aspects of current solar theory, even though I don't think it actually "fits" with the actual satellite observations, the heliosiesmology data, the nuclear chemistry analysis, etc.&nbsp; It's only the "magnetic reconnection" aspect of current solar theory that I have a hard time with, only because it's never been demonstrated in controlled conditions. </p><p>Replying to:<BR/><DIV CLASS='Discussion_PostQuote'>Why don't you realize why this makes your accusations of intellectual dishonesty on the part of mainstream astronomers so grating?&nbsp; You are doing precisely what you accuse them of doing.&nbsp; (Which, incidentally, is *not* what they are actually doing.</DIV></p><p>I think you're missing a critical difference in our approach.&nbsp; It is not "intellectually dishonest" to be wrong about an idea.&nbsp; It is not intellectually dishonest to discuss hydrogen fusion as an energy source because we can generate energy in the lab from hydrogen fusion. I therefore has no particular beef with standard solar theory with the single exception of "magnetic reconnection". &nbsp; </p><p>Replying to:<BR/><DIV CLASS='Discussion_PostQuote'>What they are doing is much more involved.&nbsp; However, now that I know your professional background is similar to mine, perhaps that makes sense.&nbsp; Software engineers don't have a whole lot of perspective into the daily workaday life of an astrophysicist.)</DIV></p><p>Perhaps not, but I have been talking with them regularly now for many years. I ask a lot of them for instance if they've ever read the book "Cosmic Plasma" by Hannes Alfven.&nbsp; None of the astronomers that I've talked with in cyberspace have ever read that book, yet the mainstream astronomers gave him a Nobel Prize, and that book represents the basis of plasma physics and plasma cosmology theory.&nbsp; It's hard for me to imagine how a "rounded" astronomy education would not include that book somewhere in the cirriculum. &nbsp; I can't peer into their daily lives, but I can certainly gain some insight intto some of the problems plaguing the industry. </p><p>I certainly do see a significant distinction between an astronomer that works with neutrinos and one that writes about "inflation".&nbsp; I can test any theory related to neutrinos.&nbsp; I can't test anything related to inflation.&nbsp; That's the difference.&nbsp;</p><p>Replying to:<BR/><DIV CLASS='Discussion_PostQuote'>Regarding sunspots, do you think they are dark?&nbsp; They aren't.&nbsp; Only laypeople think they're dark.&nbsp;&nbsp; They're actually extremely luminous.&nbsp; It's just that the surrounding sun is more luminous, so by comparison, they seem dark.</DIV></p><p>I agree. There is however a distinct difference between the amount of light we see in the umbra, vs the light we observe on the rest of the surface of the photosphere. &nbsp; They key observation however is the fact that the sides of the penumbral filaments end at a very specific depth.&nbsp; The photosphere is not made of the same material as the material underneath the photosphere which is why we see that the light does not extend along the sidea of the penumbral filaments beyond a particular depth. &nbsp; This is not unlike looking down into the eye of a tornado and seeing the blue sky in the center.&nbsp; You can see beneath the plasma cover in sunspots to the plasma below the photosphere.</p><p>Replying to:<BR/><DIV CLASS='Discussion_PostQuote'>Watching sunspots through a telescope is fascinating.&nbsp; There was a really big one a couple of years ago, and it was fascinating watching it.&nbsp; You could see the magnetic field lines, plain as day, in the little fringes around the edges.&nbsp; It was just exactly like metal filings around the poles of a magnet. </DIV></p><p>Agreed, it is very fascinating to watch.&nbsp; FYI, those magentic field lines are directly related to current that is flowing through the plasma.&nbsp; They are not simply "magnetic fields", they are "Birkeland currents." </p><p>Replying to:<BR/><DIV CLASS='Discussion_PostQuote'>You can see it in the TRACE imagery too, which has awesomely detailed videos of coronal loops.&nbsp; They look exactly like magnetic field lines.</DIV></p><p>Well, they are in fact moving magnetic field lines composed of moving charged particles (current flow).&nbsp; There is clearly a magnetic field around these loops, but that field is sustained by the movement of the charged particles inside the loops. The give away that there electron current flow involved is the fact they reach tens of millions of degrees Kelvin. &nbsp; We can easily explain that behavior with elecrtrical current. &nbsp; It's not a coincidence that Rhessi observes gamma rays from the Earth's atmosphere and gamma rays from the sun's atmospehre.&nbsp; On Earth we have isolated these events to electrical discharges in the Earth's atmosphere.</p><p>Replying to:<BR/><DIV CLASS='Discussion_PostQuote'>I still think you've got a very confused impression of what "magnetic reconnection" means. </DIV></p><p>Of course I am confused.&nbsp; What else is possible when no one has ever defined the physical process that releases energy in "magnetic reconnection"?&nbsp; I have no way to determine what that statement really means, particularly as it relates to solar wind movement that is uniformly outbound in all directions.&nbsp;&nbsp; A charge attraction between the photosphere and the heliopshere would explain that.&nbsp; Magnetic reconnection as far as I know is simply a misconcieved idea based on a misperception about the properties of plasma.&nbsp; Alfven certainly thought this was the case.&nbsp; I certainly have never seen anyone sustain particle movement for years at time using "magnetic reconnection".</p><p>Replying to:<BR/><DIV CLASS='Discussion_PostQuote'>You seem to be using the term to describe a complete model of the Sun, the solar wind, and the Earth's magnetosphere, but the scientists who study the phenomenon are not using the term that way themselves.&nbsp; They are using it to refer to a specific sort of event, not to the entire system. And that may be where the confusion is arising. </DIV></p><p>When someone at PPL writes on their website that solar wind, and Earth's aurora are caused by "magnetic reconnection", it's hard not to be confused.&nbsp; They seem to be using the term as a gap filler for everything they cannot otherwise explain without resorting to electrical current.&nbsp; Birkeland created aurora and coronal loops around spheres in his lab and sustained them using a flow of electrons as the energy source. &nbsp; No one ever did that with "magnetic reconnection".&nbsp; Alfven rejected the whole concept outright.&nbsp; Of course I'm confused then that someone would claim that aurora and charged particle movement is caused by "magnetic reconnection", particularly when no physical definition of "magnetic reconnection" has ever been put forth.&nbsp;&nbsp; It's confusing behavior on their part. </p><p>Replying to:<BR/><DIV CLASS='Discussion_PostQuote'>Certainly it's making it difficult to understand what exactly your objections are to mainstream theories, since so far you seem to be objecting to things which are not claimed by mainstream theory -- such as that energy is created from nothing during magnetic reconnection, which is not claimed at all.</DIV></p><p>What is the claim then?&nbsp; Why does WIKI claim it's a violation of energy conservation?&nbsp; How does "magnetic reconnection" differ from particle reconnection or electrical reconnection?&nbsp;</p><p>Replying to:<BR/><DIV CLASS='Discussion_PostQuote'>When they say energy is released, that implies that energy was stored in some other form, not that it was created out of nothing.</DIV></p><p>A "magnetic reconnection" energy release event *without* a current sheet would in fact violate the laws of physics as we understand them, paricularly in light plasma.&nbsp; As I said, this subject is very confusing because no one has done the ground work on the physics behind "magnetic reconnection".&nbsp; As far as I can tell, the only 'reconnection" that ever goes on in plasma is particle reconnection, and electrical reconnection. &nbsp; Magnetic fields always form a full continuum. They don't make and break connections like electrical circuits and charged particles, and the sun's photosphere is composed of mostly charged particles in motion (current flow).&nbsp;&nbsp; The logical explanation for million degree plasma, and continuous acceleration of solar wind is electricity.&nbsp; There is no real physics layed out to explain "magnetic reconnection" so it's impossible to tell if such a process even exists in nature. </p><p>Replying to:<BR/><DIV CLASS='Discussion_PostQuote'>BTW, when I asked where you could possibly have gotten your "96% of astronomy is metaphysics" statistic, I kinda suspected you'd answer "oh, that's the percentage of dark matter".&nbsp; But that's silly.&nbsp; You were expressing that statistic as a percentage of astronomical empiricism, not as a percentage of how much matter is presently accounted for.&nbsp; Dark matter is certainly not the only thing astronomers study; far from it.&nbsp; More astronomers study things like periodic comets and asteroids and extrasolar planets and planetary nebulas and things like that, which means that the 96% figure is referring to a supposed deficiency in a minority of the discipline, thus grossly distorting the numbers even if the 96% really did refer to percentage of wooiness rather than percentage of matter which has been detected by means other than its gravitational influence. <br /> Posted by CalliArcale</DIV></p><p>As best as I can tell, "dark" terms like "dark energy" and "dark matter" are nothing more than placeholder terms for human ignorance.&nbsp; These metaphysical gap fillers have not been physicallly defined or identified, yet they represent the vast majority of the objects and forces that supposedly shape our universe. &nbsp; Dark matter is but a gap filler for what amounts to "missing mass".&nbsp;&nbsp; "Dark energy" is a gap filler to explain the observed acceleration of a mostly plasma universe.&nbsp; EM fields can easily explain plasma acceleration, so I see no need for "dark energy" to explain an observation of acceleration.</p><p>I certainly do hear your point however, and I see what you mean.&nbsp; I have personally benefitted from the work of many astronomers, and it's really not fair of me to condemn the entire industry for the sins of a few.&nbsp; I really do hear you on that point.&nbsp;&nbsp; I do.</p><p>On the other hand, I can't help but see some serious and systemic problems in an industry that puts unwavering faith in "dark" things, inflation, monopoles, magnetic reconnection and metaphysical entities galore rather than putting it's faith in emprical testing and plasma phyiscs.&nbsp;&nbsp; As you can see from this thread, anyone who posits ideas which don't coincide with the current dogma of astronomy is met with comments about "mental problems" and personal attacks.&nbsp; Never mind the fact that "dark energy" and inflation are new dogmas of the industry and were not even discussed during my college days.&nbsp; </p><p>Obviously there are people like you that are open minded and more scientific in their approach to astronomy, but as you can see for yourself, there is an extremely ugly side to questioning the dogma of astronomy.&nbsp; It shouldn't be that way.&nbsp; There should be room for honest dissent without risk of being personally attacked.&nbsp; Here is room for honest scientific dissent. but in most astronomy forums and in must publishing channels, that certainly is not the case.&nbsp; Instead of addressing the issues via science, it is far more likely that one will be banned, bad mouthed, and have their santity questioned.&nbsp; The ad hominem seems to be the weapon of choice for defending the dogma of astronomy.&nbsp; This is certainly not the sort of thing you see in the computer hardware or software fields, where competition drives the process. &nbsp;&nbsp; From a programmers perpective, this type of behavior seems almost cult like in it's approach.&nbsp; The only thing that gives me hope for the industry is that I have run into astronomy websites like this one that are real science forums, with real scientific open minded moderators.&nbsp; That part is encouraging to me.&nbsp; Two individuals should be able to disagree with each other scientifically, and still be friends, and still have respect for one another. &nbsp; Such astromomy forums do exist, but they are the rare exception in this industry. &nbsp; </p><p>&nbsp;</p> <div class="Discussion_UserSignature"> It seems to be a natural consequence of our points of view to assume that the whole of space is filled with electrons and flying electric ions of all kinds. - Kristian Birkeland </div>
 
M

michaelmozina

Guest
<p><BR/>Replying to:<BR/><DIV CLASS='Discussion_PostQuote'>Boy, you just don't believe in half measures do you Micheal? </DIV></p><p>Well, sometimes. :)&nbsp; Unfortunately EU theory and Birkeland solar theory tend to be an "all or nothing" sort of thing.&nbsp; It's like being a 'little bit pregnant.' :)&nbsp; Once you start seeing the universe that way, you can't go back.</p><p>Replying to:<BR/><DIV CLASS='Discussion_PostQuote'>Okay, I'm going to try to concisely address your posts, and ask you a few key questions. One of my first questions is: Considering just the photospheric layer, and it's spectral emissions, how do you go about calculating the various elemental populations?</DIV></p><p>It's a lot tricker to be sure since it's a mass separated model and a heat separated model as well.&nbsp; I'm not sure there is any logical way to use specrometry alone to determine elemental composition which is why I tend to favor Dr. Manuel's approach.&nbsp;</p><p>As it relates to elemental layers, in theory, a spacecraft in close proximity to the sun, with the right filters should be able to pick out the varoius layers.&nbsp; Unfortunately most solar satellites are pretty far away from the sun with iimited resolution and with limited filtering capability.&nbsp; Most of them are "monocular" systems with the exception of STEREO.&nbsp; STEREO's filters are most sensitivive to iron ion wavelengths, and helium wavelenths.&nbsp; It doesn't for instance have a Neon specific wavelenth filter.</p><p>Replying to:<BR/><DIV CLASS='Discussion_PostQuote'>I'm asking for a brief run down here, such as which equation do you use, what are some things you have to keep in mind. Depending on that answer, I'll consider any compositions you derived from spectra. Otherwise I'll tell you where you went wrong. </DIV></p><p>&nbsp;http://arxiv.org/abs/astro-ph/0609509</p><p>Replying to:<BR/><DIV CLASS='Discussion_PostQuote'>This approach, btw, is backed by empirical data in our own labs. Now, as for the seperation of atoms in the sun. Okay, I'll buy that. Heck, I'll give it to you! </DIV></p><p>You are welcome to read the paper before you "give" me anything you might later regret. :)</p><p>Replying to:<BR/><DIV CLASS='Discussion_PostQuote'>The standard model already calls for it! Only the standard model doesn't have seperation throughout the entire sun. The core can be heavily seperated. However due to the temperature gradients in the sun, dictated by the fact that it's in hydrostatic equilibrium, and the required energy transportation through the sun to maintain that state...we have a region of the sun with very strong convective currents. This is using the same appraoch to deterimine the flow of boiling water btw. In the presence of such convective currents how do you prevent mixing? </DIV></p><p>Well, this brings us to that problem of that "stratification subsurface" sitting smack dab in th middle of what is supposed to be an open convection zone.</p><p>http://arxiv.org/abs/astro-ph/0510111</p><p>Now it so happens that plasma downflows in sunspots stop their downward path and flatten out into a horizontal flow around that location as well.&nbsp; What's a stratification subsurface doing in what is supposed to be an open convection zone?&nbsp; How do you know that stratification subsurface isnt allowing for more mass separation than you predicted with the standard model?</p><p>Could you briefly explain "coronal rain" from your perspective?</p><p>Replying to:<BR/><DIV CLASS='Discussion_PostQuote'> ------------------- The fact that the outer part of the atmosphere is mostly made of hydrogen and helium, and the fact these are the hottest layers of the atmosphere causes these elements to emit and block the most photons. The spectral data will necessarily be skewed in that direction. ------------------- Here you demonstrate that you have a bit of a clue as to some of the problems in spectroscopy. You correctly indicate that the temperature of the atoms can change how much light they absorb. But you've got something wrong here. You can be to hot, where vast numbers of the atoms are actually stripped of their electrons...and can no longer intercept as much (or in some cases ANY light). here's a small secrete: The suns spectra is heavily dominated by numerous, deep adn dark IRON lines (and other "metals" like calcium), and relatively few helium and hydrogen lines. -----------------------</DIV></p><p>We can use satellite technologies like SOHO and TRACE to isolate the bulk of the iron ion lines. &nbsp; Not surprisingly, the bulk of these emissions come from the coronal loops, not the surface of the photosophere.&nbsp; We can also isolate hydrogen and helium emissions to some degree which certainly are more uniformly distributed around the sun.&nbsp; If there was a uniform distribution of iron emissions coming from the whole sun like we see in hydrogen and helium filters, then a "composition based on photon output" might fly.&nbsp;&nbsp; The sun is however a much more complicated system that it might appear at first glance.&nbsp; Any oversimplied method of calculating it's composition is doomed to failure. </p><p>Replying to:<BR/><DIV CLASS='Discussion_PostQuote'>I looked at the coronal loops and assumed they contained and emitted most of the iron and nickel ion photons, I assumed the crust contained most of the heavy elements, and I arranged the rest of the SERTS elements by atomic weight. This implies that the outer layer (corona) is mostly hydrogen, the chromosphere is mostly helium, the photosphere is mostly neon, and the layer under the photosphere is mostly silicon. Under that probably sits a layer of calcium as well. ------------------------ First, why are you assuming all this? Look at the spectra and find out.</DIV></p><p>I began with the SERTS spectral data.&nbsp; That data alone did not tell me where the various ion photons were coming from.&nbsp; To know where they were coming from, I used satellite technology to isolate the metal lines.&nbsp; They were *not* uniformly distributed around the sun like I found with helium lines or hydrogen lines.&nbsp; The natural question I asked is "why"? The answers to the "why" part led the various assumptions I made.&nbsp; I looked at coronal rain.&nbsp; I looked at running difference images and Nickel ion Doppler images. I used a lot of bits of information to put together a cohesive model.&nbsp; Keep in mind that all of this was done *before* I found the work of Kristian Birkeland or Oliver Manuel. &nbsp; I based my opinions primarily on SERTS data and satellite (and ground) imagery.</p><p>Replying to:<BR/><DIV CLASS='Discussion_PostQuote'>Also, you say that the coronal loops emit mostly iron and nickel spectra, but is mostly hydrogen... ????????</DIV></p><p>Well, a coronal loop is highly electrified.&nbsp; It's very energetic compared to say the 6K photosphere.&nbsp; That current flow heats the plasma inside the loops to millions of degrees, temps that are required to emit some of the spectral lines we find in the SERTS data.&nbsp; Most of the metal lines come from the coronal loops.</p><p>Replying to:<BR/><DIV CLASS='Discussion_PostQuote'>At what point does the sun stop being predominately hydrogen and helium? I'm looking for a "depth" figure here. ????????</DIV></p><p>Keep in mind that hydrogen flows through all the layers and is constanly being created in coronal loop activity as neutrons are pinched from the plasma and decay into protons and electrons and neutrinos.&nbsp; The sun is also very energetic and elements are flying all over the place.&nbsp;&nbsp; All the layers are separated, but it's a very actively mobile environment. </p><p>The bulk of the hydrogen sits above the chromosphere in the corona and it's being accelerated toward the heliosheath on an ongoing basis.&nbsp; I used the standard depth of the chromosphere and photosphere to determine the depth of the hydrogen and neon layars respectively.&nbsp; The Silicon and Calcium depths came from heliosiesmology data. &nbsp; I'm at work right now but I'll post the depth's later today.&nbsp; I probably have them posted in the original solar thread, but I can't recall them off the top of my head at the moment.&nbsp; &nbsp; </p><p>Replying to:<BR/><DIV CLASS='Discussion_PostQuote'>You refer to a satelite image nobody's been able to explain...what is the image, I'd love to take a look at it. I see even more references to "overabundance of hydrogen and helium" when that isn't the case. What spectra are you looking at? <br /> Posted by Saiph</DIV></p><p>http://www.thesurfaceofthesun.com</p><p>Take a look at that first gold RD image from TRACE at 171A.&nbsp;&nbsp; I believe you will find the links to these images on the LMSAL site on my first page. &nbsp;&nbsp;</p><p>I'd like to start our analysis of the image by agreeing upon the light source of these images and the "dust' (heavy plasma) we see blowing around in the image right after the CME event.&nbsp; </p><p>&nbsp;</p> <div class="Discussion_UserSignature"> It seems to be a natural consequence of our points of view to assume that the whole of space is filled with electrons and flying electric ions of all kinds. - Kristian Birkeland </div>
 
S

Saiph

Guest
<p>Okay, I don't have tons of time at this particular moment, so I'll elaborate later.</p><p>You do realize that spectral lines are HIGHLY temperature dependant right?&nbsp; That in the really hot coronal areas, Hydrogen and Helium produce&nbsp; almost ZERO spectral lines because they are completely ionized? That any lines you do find there are from the few atoms that just happen to be in the right state, at the right time...before they are re-ionized.&nbsp; So you have to scale the strength of those lines by factors of millions.</p><p>I was hoping you'd mention this, and the "SAHA-BOLTZMAN" equation.&nbsp; The fact that this didn't crop up at all means you really need to buckle down and learn how the figures are arrived at before you go haring off into the alternative science.</p><p>&nbsp;</p><p>Also, on Dr. Manuels theories.&nbsp; He states that the sun is formed on a neutron star from a super nova.&nbsp; He doesn't deny, and in fact relies, on the validity of the standard model for other stars (thus the supernova).&nbsp; He just doesn't think the sun is normal.&nbsp; I've asked him.&nbsp; He won't say other stars don't work like astronomers say they do, just the sun.&nbsp;</p> <div class="Discussion_UserSignature"> <p align="center"><font color="#c0c0c0"><br /></font></p><p align="center"><font color="#999999"><em><font size="1">--------</font></em></font><font color="#999999"><em><font size="1">--------</font></em></font><font color="#999999"><em><font size="1">----</font></em></font><font color="#666699">SaiphMOD@gmail.com </font><font color="#999999"><em><font size="1">-------------------</font></em></font></p><p><font color="#999999"><em><font size="1">"This is my Timey Wimey Detector.  Goes "bing" when there's stuff.  It also fries eggs at 30 paces, wether you want it to or not actually.  I've learned to stay away from hens: It's not pretty when they blow" -- </font></em></font><font size="1" color="#999999">The Tenth Doctor, "Blink"</font></p> </div>
 
M

michaelmozina

Guest
<p><BR/>Replying to:<BR/><DIV CLASS='Discussion_PostQuote'>Okay, I don't have tons of time at this particular moment, so I'll elaborate later.You do realize that spectral lines are HIGHLY temperature dependant right? </DIV></p><p>Of course.&nbsp; That is why I specifically went looking for the iron ion photons (which require lots of heat) were located in the satellite imagery. &nbsp; The coronal loops are where the all million degree "action" is happening.&nbsp; That is where we find the iron lines are concentrated.&nbsp;</p><p>Replying to:<BR/><DIV CLASS='Discussion_PostQuote'>That in the really hot coronal areas, Hydrogen and Helium produce&nbsp; almost ZERO spectral lines because they are completely ionized? That any lines you do find there are from the few atoms that just happen to be in the right state, at the right time...before they are re-ionized.&nbsp; So you have to scale the strength of those lines by factors of millions.I was hoping you'd mention this, and the "SAHA-BOLTZMAN" equation.&nbsp; The fact that this didn't crop up at all means you really need to buckle down and learn how the figures are arrived at before you go haring off into the alternative science.</DIV></p><p>Um, you asked for an overview, and I'm at work right now.&nbsp; I didn't try to give you the verbose explanation in the very first post. :)&nbsp; Don't get carried away about assuming anything this early in our conversation.&nbsp; I was mostly trying to give folks the overview of what to expect from a Birkeland, mass separated solar model, and to address some of the key points. </p><p>Replying to:<BR/><DIV CLASS='Discussion_PostQuote'>Also, on Dr. Manuels theories.&nbsp; He states that the sun is formed on a neutron star from a super nova. </DIV></p><p>And there has been confirmation of a recent supernova in our vicinity, shortly before our sun was born.&nbsp;&nbsp; </p><p>Replying to:<BR/><DIV CLASS='Discussion_PostQuote'>He doesn't deny, and in fact relies, on the validity of the standard model for other stars (thus the supernova).&nbsp; He just doesn't think the sun is normal.&nbsp; I've asked him.&nbsp; He won't say other stars don't work like astronomers say they do, just the sun.&nbsp; <br /> Posted by Saiph</DIV></p><p>I think it's been awhile since you've talked with him then.&nbsp; I'm sure his positions have changed since finding my work and since getting involved in EU theory.&nbsp; I for one do not believe that our sun is particularly unique in design, and I"ve never heard him suggest that it is.&nbsp; I'm technically a "Johnny come lately" member of the iron sun theory however, so I may have missed some of your earlier conversations.&nbsp; My introduction to his work came about 3 year ago.&nbsp; When did you have your conversation with Dr. Manuel? </p><p>&nbsp;</p> <div class="Discussion_UserSignature"> It seems to be a natural consequence of our points of view to assume that the whole of space is filled with electrons and flying electric ions of all kinds. - Kristian Birkeland </div>
 
M

michaelmozina

Guest
<p><BR/>Replying to:<BR/><DIV CLASS='Discussion_PostQuote'>What I have gotten out of your posts on this thread and the previous one is:The Sun like the other stars in the universe is primarily powered, not by fusion, but by a current that flows through the cosmos, motive source unknown, </DIV></p><p>Actually Alfven positied some theories about the motive force, but I personally tend to prefer to simply note that the current flow exists and leave it at that. Suffice to say, there are ideas about the source that have been put on the table by Alfven and others. </p><p>Replying to:<BR/><DIV CLASS='Discussion_PostQuote'>Since we do not see the enormous magnetic field at the surface of the Earth that would result fron any such current, one must conclude that Maxwell's equations are very much in error.</DIV></p><p>Were it not for the fact we are isolated from this current by the heliosphere and magnetopshere, you might have a point. Instead, one must conclude that Maxwell's equations are very accurate and that they apply to the activities of the universe as well.</p><p>Replying to:<BR/><DIV CLASS='Discussion_PostQuote'>It is electrical effects, rather than gravitational effects, that govern the behavior of the larges masses in the universe at the largest distance scales. </DIV></p><p>If you mean I have to choose between EM fields generating the observation of acceleration of objects or "dark energy", yes, I'll choose EM fields every time.&nbsp; I don't presume that EM fields dominate our particular solar system activity however.&nbsp; Maybe they have an affect, but I'm fine with GR just as Einstein taught it (without constants).&nbsp; I'm sure gravity plays a major role in our lives and in this solar system.&nbsp; It probably plays the dominant role, and gravity is probably ultimately an EM effect in the final "grand unified theory" analysis of things.</p><p>Replying to:<BR/><DIV CLASS='Discussion_PostQuote'>Mainstream physicists are just to bliind to see this.</DIV></p><p>Some aren't blind at all. Alfven wasn't blind to it. He wrote about.&nbsp; Peratt isn't blind and he works at Los Alamos.&nbsp; I guess you'd have to call Alfven "mainstream" since he won a Nobel Prize for his work on plasma physics.&nbsp; I guess it comes down to individuals and their parituclar beliefs.</p><p>Replying to:<BR/><DIV CLASS='Discussion_PostQuote'>The photosphere of the Sun is composed&nbsp;primarily of neon, not hydrogen as is believed by the mainstream.</DIV></p><p>Yes.</p><p>Replying to:<BR/><DIV CLASS='Discussion_PostQuote'>The Sun is basically a giant neon light bulb.</DIV></p><p>It's a bit more complicated than that, but essentially yes.&nbsp; The sun is composed of current carrying plasma, including "neon".&nbsp; These plasma light up quite brilliantly due to the current that runs through them.</p><p>Replying to:<BR/><DIV CLASS='Discussion_PostQuote'>The core of the Sun is not a hot gaseous plasma, but rather is solid, primarily composed of nickel and iron or at least a nickel iron shell over a neutron core.</DIV></p><p>Assuming it has a small neutron central core, the interior must contain high temperature plasma as well.&nbsp; I can't technically observe anything below the crust so I remain open to a lot of options as it relates to the core. </p><p>Replying to:<BR/><DIV CLASS='Discussion_PostQuote'> It remains solid despite being in the interior of an extremely hot radiating sphere, which would imply an enormous radiant heat flux.</DIV></p><p>Again, you are "assumng" a non EM enviroment.&nbsp; Spheres do unsual things in space due to surface tension and EM effects.</p><p>http://www.youtube.com/watch?v=cXsvy2tBJlU</p><p>Why doesn't that water shell collapse in upon the air bubble?&nbsp; Air is obviously far less dense than the water shell.</p><p>Replying to:<BR/><DIV CLASS='Discussion_PostQuote'>Magnetic fields are not particularly important to the behavior of plasma at and near the surface of the Sun, but rather it is electrical currents and lightning-like discharges that govern.</DIV></p><p>Suggesting that magnetic fields are not important is misleading and it's your own strawman.&nbsp; All the forces are important, but the electron flow drives the process.</p><p>Replying to:<BR/><DIV CLASS='Discussion_PostQuote'>Topological changes in the magnetic field cannot and do not exist,</DIV></p><p>Magnetic fields do not "magnetically reconnect".&nbsp; Particles reconnect and charges reconnect, but magnetic fields always form a full continuum. </p><p>Replying to:<BR/><DIV CLASS='Discussion_PostQuote'>and modern physicists studying the topic are deluded.</DIV></p><p>Which physicsts are you talking about?&nbsp; Peratt is not deluded.&nbsp; Alfven was not deluded.&nbsp; Which physicist were you refering to specifcially?&nbsp; It sounds to me like you're just grandstanding at this point. </p><p>Replying to:<BR/><DIV CLASS='Discussion_PostQuote'>Because physicists and engineers use diagrams with field lines to describe fields, not having at their disposal the necessary 4-dimensional paper to depict them accurately, they don't understand the continuous nature of vector fields.</DIV></p><p>Er, no.&nbsp; I just believe Alfven was correct about "magnetic reconnection" theory.&nbsp; Got an empircal test to demonstrate he was wrong on this point?</p><p>Replying to:<BR/><DIV CLASS='Discussion_PostQuote'>One can only conclude that they have no grasp of the behavior of either magnetic fields or of plasmas.</DIV></p><p>I never suggested such a thing.&nbsp; I only said that magnetic reconnection was a false concept based on a misconception abou the behaviors of plasma.&nbsp; Not every physicist in the world believes in "magnetic reconnection".&nbsp; Alfven certainly did not.&nbsp; Peratt does not.&nbsp; It's not me vs. the physicst of the world.&nbsp; It's physicist vs. physicist.&nbsp; I'm just an innocent bystander who puts more faith in the guy that wrote plasma physics theory than the guys claiming that magnetic reconnection is a unique form of energy exchange.</p><p>Replying to:<BR/><DIV CLASS='Discussion_PostQuote'>Modern physicists, the ongoing experiments at the Princeton Plasma Physics lab notwithstanding, do not have any experimental evidence for their hypotheses regarding magnetic reconnection.</DIV></p><p>No, in fact they do not.&nbsp; They never isolated the speicific energy release mechanism behind "magnetic reconnection" so there is no physical way to differentiate "magnetic reconnection" from "particle reconnection" or "eletrical reconnection". &nbsp; Don't blame the messenger.&nbsp; I would suggest that at least they are going about the issue the right way, in a lab, in controlled experimentation.&nbsp; Their failure to find the physics of "magnetic reconnection" is not their fault IMO, but rather the the failure is due to a fundamental flow in the theory they are studying.&nbsp;&nbsp; I really don't have any problem with their research either.&nbsp; Active experimentation is the only way to disprove Alfven.&nbsp; They just haven't done it.&nbsp; It's only when astronomers point to the sky and claim "magnetic reconnection did it" that I cry foul.</p><p>Replying to:<BR/><DIV CLASS='Discussion_PostQuote'>The 100 year old work of Birkeland is better and more accurate than what is being done today with far more sophisticated equipment and substantial funding.</DIV></p><p>Without any ultra sophisticated equipment Birkeland ws able to sustain aurora and coronal loops around sphere using electricity over 100&nbsp; years ago.&nbsp;&nbsp; Mind you it was sophisticated gear for it's time, but by today's standards it was rather primative.&nbsp; Why can't PPL do that using only "magnetic reconnection"?&nbsp; Is an empircal test of concept really too much to ask for from a bunch of guys with all that funding and fancy equipment and 100 years of technological progress?&nbsp; Birkeland did all this.&nbsp; It&nbsp; was done over 100 year ago by one guy using electricity. What's the big deal?&nbsp; Why would you not wish to see this demonstrated with "magnetic reconnection"?</p><p>Replying to:<BR/><DIV CLASS='Discussion_PostQuote'>Physicists and astronomers do not understand "electrical theory" as only an electrical engineer can, </DIV></p><p>Lot's of physicists I've talked with seem to understand it just fine.&nbsp; Some astronomers too.&nbsp; Most of the astronomers I meet are pretty clueless when it comes to electrical theory since it's not a prerequisite for a degree in astronomy.&nbsp; Oddly enough I've never met an electical engineer that puts any faith at all in "magnetic reconnection".&nbsp; Most of them I've talked with felt pretty much like Scott and Alfven and Peratt.</p><p>Replying to:<BR/><DIV CLASS='Discussion_PostQuote'>despite the fact that physics and engineering texts on electromagnetism say the same things in pretty much the same way.</DIV></p><p>Aflven and Scott don't see things the same way as the folks at PPL.&nbsp; Now what? Do we simply ignore that point or what?</p><p>Replying to:<BR/><DIV CLASS='Discussion_PostQuote'>Mainstream scientists do not give adequate credance to EU theory or solar theory and conspire to prevent its publication, despite the obvious merits. </DIV></p><p>Yep. That is my claim.&nbsp; You could refute that claim extremely easily with a few EU articles from the APJ.&nbsp; The IEEE has no trouble at all finding suitable EU matieral to publish.&nbsp; What is the problem with the APJ? Why does BAUT have a completely different rule system for "inflation" than for "EU theory"?&nbsp;&nbsp; </p><p>Replying to:<BR/><DIV CLASS='Discussion_PostQuote'>The several papers that have evaluated the premises of EU theory and have found them wanting on scientific grounds should be ignored, as the physicists just don't understand. </DIV></p><p>I have no idea what statement is supposed to mean. I simply noted that the APJ publishes *zero* material on EU theory and astronomy oriented websites often apply unique and rather draconian rules to the topic of EU theory.&nbsp; You're sort of making up your own strawmen now as best as I can tell.&nbsp; </p><p>Replying to:<BR/><DIV CLASS='Discussion_PostQuote'>Work such as yours in EU theory and solar physics ought not be ignored.</DIV></p><p>My personal work in astromomy isn't very important actually. EU theory on the other hand is *very* important,&nbsp; It certainly should not be ignored or descriminated against by the mainstream.&nbsp; If they can read and write about inflation and dark energy, then they should surely read and write about plasma physics applied to cosmology.</p><p>Replying to:<BR/><DIV CLASS='Discussion_PostQuote'>You have the answers to the riddles of the universe.</DIV></p><p>I have some answers to some riddles that continue to baffle mainstream astronomers, thanks to the hard work of Kristian Birkeland, Charles Bruce, Hannes Alfven, Anthony Peratt. Halton Arp, and many others.&nbsp; I'm not the only one with these answers however and it's not an ego thing for me since Birkeland beat me to this solar model and the EU theory by over 100 years.&nbsp; It would be silly to simply ignore their work because you personally don't care for it.&nbsp; I don't much care for MOND theory either, but I would not suggest it have special rules applied to it, or that it be excluded from publication.</p><p>Replying to:<BR/><DIV CLASS='Discussion_PostQuote'>All I can say is that I do believe that this sort of thing has received a good deal of consideration in the mainstream of academia.&nbsp; Look up Abnormal Psychology.&nbsp; <br /> Posted by DrRocket</DIV></p><p>Do the cheap shots make you feel better somehow?&nbsp; They look like an act of pure desparation from my vantage point.&nbsp; Creationists often get mad at me for pointing out the flaws in their logic, but I've learned to develop at thick skin over the years. &nbsp; Astronomers tend to be just as touchy when one points out that inflation never did anything to any object in any controlled experiment and it doesn't emprically exist.&nbsp; Somehow that fact makes me the target of personal attack. I don't take it personally anymore.&nbsp;&nbsp; I realize that some people react emotionally to having their belief systems questioned. A thick skin comes with the territory.</p><p>A real "scientist" would simply say "Michael, I know you think that the LMSAL RD image and that Doppler image show evidence of a "rigid crust" under the photosphere. but let me explain exactly what is really going on in that image......"</p><p>In three years of online debates with astromers from all over the planet, that has never happened. &nbsp; I doubt you'll do it either.&nbsp;&nbsp; The personal attacks will never sway my scientific perspective.&nbsp; Only a "better" scientific explanation could ever do that.&nbsp;&nbsp; For me at least it's totally impersonal and it's all about emprical science.&nbsp; If you have the scientific knowledge to explain those satellite images, by all means, please do so.&nbsp; If not, no amount of personal attacks will sway me from my opinions.&nbsp;&nbsp; I'll be happy to discuss the science, but let's stay focused on the science and lets keep things above the belt.</p> <div class="Discussion_UserSignature"> It seems to be a natural consequence of our points of view to assume that the whole of space is filled with electrons and flying electric ions of all kinds. - Kristian Birkeland </div>
 
S

Saiph

Guest
<p><BR/>Replying to:<BR/><DIV CLASS='Discussion_PostQuote'>Of course.&nbsp; That is why I specifically went looking for the iron ion photons (which require lots of heat) were located in the satellite imagery. &nbsp; The coronal loops are where the all million degree "action" is happening.&nbsp; That is where we find the iron lines are concentrated. </DIV></p><p>Well, you will certainly find ionized iron there...but that doesn't mean those loops are mostly iron.&nbsp; It will tell you that the iron that is there is ionized...okay, and that there is iron there.&nbsp; Okay.&nbsp; I fail to see your point though.&nbsp;</p><p>Replying to:<BR/><DIV CLASS='Discussion_PostQuote'>Um, you asked for an overview, and I'm at work right now.&nbsp; I didn't try to give you the verbose explanation in the very first post. :)&nbsp; Don't get carried away about assuming anything this early in our conversation.&nbsp; I was mostly trying to give folks the overview of what to expect from a Birkeland, mass separated solar model, and to address some of the key points. </DIV>&nbsp; Well, you certainly said a lot of things, without actually indicating any of the standard tried and true methodology.&nbsp; I'll let this slide for now.&nbsp; I'm probably going to start a "Standard model" thread here in the next few days, to explore how mainstream science models the sun (and you're welcome to ask questions in it, to&nbsp; understand that, just no EU) </p><p>&nbsp;</p><p>Replying to:<BR/><DIV CLASS='Discussion_PostQuote'>And there has been confirmation of a recent supernova in our vicinity, shortly before our sun was born.&nbsp;&nbsp; I think it's been awhile since you've talked with him then.&nbsp; I'm sure his positions have changed since finding my work and since getting involved in EU theory.&nbsp; I for one do not believe that our sun is particularly unique in design, and I"ve never heard him suggest that it is.&nbsp; I'm technically a "Johnny come lately" member of the iron sun theory however, so I may have missed some of your earlier conversations.&nbsp; My introduction to his work came about 3 year ago.&nbsp; When did you have your conversation with Dr. Manuel? &nbsp; <br /> Posted by michaelmozina</DIV><br /><br />Last I talked with him was in 2001, duringone of his talks at the MARAC conference.&nbsp; There's a significant problem with him braodening out his ideas though.&nbsp; If stars like our sun (which is amazingly average in all known regards compared to other G2 stars) was formed on the remnant of a supernova, how do stars that go supernova work?&nbsp; Are those normal hydrogen rich stars?&nbsp; For the record, I'm willing to listen to ideas about a few rare and specific stars being formed in this fashion...but all stars?&nbsp; Even all G2 stars...&nbsp; I don't buy it.&nbsp; Throw on the fact that Neutron stars are HOT and will push away any nebulae gases that wish to form around it and...well, I don't think so.</p><p>&nbsp;</p><p>I've looked at the "surfaceofthesun.com" web page, and you're refering to the first picture right?&nbsp; Okay, what about it?&nbsp; I see a localized area that isn't changing much, at high temperatures which produce lots of iron ion lines.&nbsp; These features are comparable to the size of continents or entire planets...so I don't expect them to change much in short time periods (like say, months!).&nbsp; Sorta like the big red spot on jupiter.&nbsp; It's huge, and for any feature to be that big it's either a) Insanely powerful and concentrated, and will disappear in short time frames, like an explosion.&nbsp; or b) metastable and self-building/sustaining.&nbsp; It'll take a long time for the feature to fade. &nbsp;&nbsp;</p> <div class="Discussion_UserSignature"> <p align="center"><font color="#c0c0c0"><br /></font></p><p align="center"><font color="#999999"><em><font size="1">--------</font></em></font><font color="#999999"><em><font size="1">--------</font></em></font><font color="#999999"><em><font size="1">----</font></em></font><font color="#666699">SaiphMOD@gmail.com </font><font color="#999999"><em><font size="1">-------------------</font></em></font></p><p><font color="#999999"><em><font size="1">"This is my Timey Wimey Detector.  Goes "bing" when there's stuff.  It also fries eggs at 30 paces, wether you want it to or not actually.  I've learned to stay away from hens: It's not pretty when they blow" -- </font></em></font><font size="1" color="#999999">The Tenth Doctor, "Blink"</font></p> </div>
 
D

derekmcd

Guest
<p><strong>"Actually Alfven positied some theories about the motive force, but I personally tend to prefer to simply note that the current flow exists and leave it at that. Suffice to say, there are ideas about the source that have been put on the table by Alfven and others."</strong></p><p>&nbsp;</p><p>Isn't that a bit hypocritical when you chastize mainstream cosmologists for positing dark energy?</p><p>&nbsp;</p><p><strong>"Spheres do unsual things in space due to surface tension and EM effects.</strong></p><p><strong>http://www.youtube.com/watch?v=cXsvy2tBJlU</strong></p><p><strong>Why doesn't that water shell collapse in upon the air bubble?&nbsp; Air is obviously far less dense than the water shell."</strong></p><p>&nbsp;</p><p>Not enough gravity to create pressure to overcome the surface tension?&nbsp; I'd be wiling to bet that, given enough time, the gas would escape.<br /> </p> <div class="Discussion_UserSignature"> <div> </div><br /><div><span style="color:#0000ff" class="Apple-style-span">"If something's hard to do, then it's not worth doing." - Homer Simpson</span></div> </div>
 
C

CalliArcale

Guest
<p><BR/>Replying to:<BR/><DIV CLASS='Discussion_PostQuote'>Fusion is a scientifically viable way to release energy.&nbsp; "Magnetic reconnection" is not.&nbsp; That is because fusion has been demonstrated in controlled conditions, whereas magnetic reconnection has not.</p><p>Posted by michaelmozina</DIV> </p><p>Well again, I think you're misunderstanding magnetic reconnection.&nbsp; Fusion doesn't just release energy -- it converts matter into energy.&nbsp; Magnetic reconnection, on the other hand, does not create energy.&nbsp; It is merely a mechanism for the transfer of energy from one form to another.&nbsp; A release of *stored* energy, as opposed to the creation of new energy.&nbsp; But I'm getting out of my depth and do not think I can adequately discuss the subject much further.&nbsp; I'm obviously failing to make my ideas understood, which is probably due to my lack of depth on the subject.&nbsp; You at least came out of EE, if I understand correctly.&nbsp; I came out of chemistry before switching over to CS, and while electricity and ionization and things like that are important in chemistry, they're not as important as they are in physics. </p><p>BTW, I don't deny that the flow of charged particles generate the field lines seen around sunspots and in coronal loops.&nbsp; I don't think mainstream astronomers do either; that's the explanation I've always heard.&nbsp; With the exception of ferromagnetism, you just don't get a magnetic field without a flow of charged particles. </p> <div class="Discussion_UserSignature"> <p> </p><p><font color="#666699"><em>"People assume that time is a strict progression of cause to effect, but actually from a non-linear, non-subjective viewpoint it's more like a big ball of wibbly wobbly . . . timey wimey . . . stuff."</em>  -- The Tenth Doctor, "Blink"</font></p> </div>
 
D

drwayne

Guest
<p><BR/>Replying to:<BR/><DIV CLASS='Discussion_PostQuote'> Fusion doesn't just release energy -- it converts matter into energy.&nbsp; Posted by CalliArcale</DIV></p><p>Well, at least if you are on the right side (oops, make that the left side) of the BE/A curve.&nbsp; :)</p><p>Wayne<br /></p> <div class="Discussion_UserSignature"> <p>"1) Give no quarter; 2) Take no prisoners; 3) Sink everything."  Admiral Jackie Fisher</p> </div>
 
S

Saiph

Guest
<p>Micheal, what about the standard model do you think DOESN"T work?</p><p>You propose an alternative...but is the standard model broken?&nbsp; Is core stellar fusion impossible?&nbsp; Why can't the sun work, as we see it, if it's made of hydrogen and helium (which, btw, do a wonderful job of accounting for the suns mass)</p><p>&nbsp;</p><p>Maybe you should just think about the question, I'll probably whip up a dedicated thread on the topic soon.&nbsp;</p> <div class="Discussion_UserSignature"> <p align="center"><font color="#c0c0c0"><br /></font></p><p align="center"><font color="#999999"><em><font size="1">--------</font></em></font><font color="#999999"><em><font size="1">--------</font></em></font><font color="#999999"><em><font size="1">----</font></em></font><font color="#666699">SaiphMOD@gmail.com </font><font color="#999999"><em><font size="1">-------------------</font></em></font></p><p><font color="#999999"><em><font size="1">"This is my Timey Wimey Detector.  Goes "bing" when there's stuff.  It also fries eggs at 30 paces, wether you want it to or not actually.  I've learned to stay away from hens: It's not pretty when they blow" -- </font></em></font><font size="1" color="#999999">The Tenth Doctor, "Blink"</font></p> </div>
 
Status
Not open for further replies.

Latest posts